LSAT and Law School Admissions Forum

Get expert LSAT preparation and law school admissions advice from PowerScore Test Preparation.

User avatar
 Dave Killoran
PowerScore Staff
  • PowerScore Staff
  • Posts: 5853
  • Joined: Mar 25, 2011
|
#45452
Complete Question Explanation
(The complete setup for this game can be found here: lsat/viewtopic.php?t=2450)

The correct answer choice is (E)

Answer choice (A) is incorrect because it would cause a violation of the fourth rule.

Answer choice (B) is incorrect because if G drives on Monday, then according to the last rule J does not drive on Saturday, and consequently, according to the fourth rule, J must drive on Wednesday.

Answer choice (C) is incorrect because if J drove on both Tuesday and Friday, then in complying with the fourth rule J would drive either Wednesday or Saturday, which would create a violation of the second rule.

Answer choice (D) is incorrect because, like answer choice (C), it would lead to a violation of the second rule. If G drives on Monday, then J cannot drive on Saturday and J must drive on Wednesday. Because the answer choice also specifies that J drives on Thursday, the second rule would be violated.

Answer choice (E) is the correct answer.

Get the most out of your LSAT Prep Plus subscription.

Analyze and track your performance with our Testing and Analytics Package.